Mathematik online lernen im Mathe-Forum. Nachhilfe online
Startseite » Forum » Untervektorräume

Untervektorräume

Universität / Fachhochschule

Vektorräume

Tags: Vektorraum

 
Antworten Neue Frage stellen Im Forum suchen
Neue Frage
lina12

lina12 aktiv_icon

23:39 Uhr, 10.02.2019

Antworten
Könntet ihr mir bitte bei dieser Aufgabe helfen?

IMG_9104

Für alle, die mir helfen möchten (automatisch von OnlineMathe generiert):
"Ich benötige bitte nur das Ergebnis und keinen längeren Lösungsweg."
Online-Nachhilfe in Mathematik
Antwort
ledum

ledum aktiv_icon

00:53 Uhr, 11.02.2019

Antworten
Hallo
wo liegen denn deine Schwierigkeiten, oft sind die weg, wenn man das erst mal mit 2×2 und 3×3 Matrizen macht, dann ist das allgemeine klar.
Gruß lul
lina12

lina12 aktiv_icon

09:03 Uhr, 11.02.2019

Antworten
Also die a) denk ich habe ich richtig aber den Rest nicht wirklich. beispielsweise bei der b) wie müsste ich da vorgehen um das zu zeigen?
Antwort
pwmeyer

pwmeyer aktiv_icon

09:32 Uhr, 11.02.2019

Antworten
Hallo,

setze mal B:=12(A+AT) und prüfe ob BU, also bestimme BT und prüfe, ob B=BT

Gruß pwm
lina12

lina12 aktiv_icon

10:19 Uhr, 11.02.2019

Antworten
Stimmt das jetzt so?:-)

IMG_9110
lina12

lina12 aktiv_icon

16:34 Uhr, 11.02.2019

Antworten
stimmt das jetzt?

IMG_9110
Antwort
pwmeyer

pwmeyer aktiv_icon

17:17 Uhr, 11.02.2019

Antworten
Hallo,

ja das stimmt.

Für den nächsten Schritt musst Du Dir die Definitionen anschauen:

Was bedeutet "Summe von Unterräumen"?
Was bedeutet darüber hinaus "direkte Summe von Unterräumen"?

Gruß pwm
Antwort
godzilla12

godzilla12 aktiv_icon

18:21 Uhr, 11.02.2019

Antworten
Diese Aufgabe habe ich schon gefühlte 4711 Mal gesehen - häufig gleich mehrfach am selben Tag .. Ich will mich mal der d) annehmen.


Die Summe V=V1+V2 zweier Vektorräume heißt direkt, wenn eine der drei folgenden äquivalenten aussagen vorliegt:


1)    V1V2={0}    (1)

( Jeder Vektorraum enthält dier Null; es soll aber ausschließlich die Null sein. )


2) Eindeutigkeit der Zerlegung:


    x1;2V1;2,    y1;2V1;2,    x1+x2=y1+y2x1=y1,    x2=y2    (2)



3)dimV=dimV1+dimV2    (3)



Aus Unterpunkt bc) deiner Aufgabe erhellt schonmal, dass


     RR^(nxn) = V_sym + V_ant     (4)


Hier warum kann der nicht sagen " hoch nXn " ; und warum kann der keine subscripts?
Hier hatt du Ahung von Füsik? Schau mal im Herbert Goldstein ( Mechanik ) oder in Becker-Sauter

" Theorie der Elektrizität "

warum dass du dich mit dieser Aufgabe beschäftigst.

" Das intressiert mich aber nicht. Ich SOLL nur diese aufgabe lösen; und ich WILL nur die neun Punkte ... "

Ich benutze Kriterium 1)( und mach das jetzt mit der Hermiteschen Konjugation; aber im Prinzip ist das egal. )


    A:= A_(sym) = A_(ant)     |  (+)    (5a)

    A=A=-A2A=0A=0;     wzbw     (5b)
Diese Frage wurde automatisch geschlossen, da der Fragesteller kein Interesse mehr an der Frage gezeigt hat.